Showing solution satisfies PDE (chain rule help)












0












$begingroup$



We have $$u(x, t) = g(x-at, 0) + int_0^t f(x+a(s-t), s)ds$$ and want to show it satisfies the PDE $$partial_t u + a cdot Du = f$$




The solution given goes as follows $$partial_t u + acdot Du = - a cdot nabla g(x-at) +f(x,t) + int_0^t nabla f(x+(s-t)a, s) cdot (-a) ds \ + a cdot nabla g(x-at) + acdot int_0^t nabla f(x+(s-t)a, s) ds$$ $$= f(x, t)$$ as required



However, I have no idea how these terms have appeared. In particular, I'm not sure why we have a grad appearing for $g$ and the $f(x, t)$ seems to come from nowhere...



Any insight is appreciated.










share|cite|improve this question











$endgroup$

















    0












    $begingroup$



    We have $$u(x, t) = g(x-at, 0) + int_0^t f(x+a(s-t), s)ds$$ and want to show it satisfies the PDE $$partial_t u + a cdot Du = f$$




    The solution given goes as follows $$partial_t u + acdot Du = - a cdot nabla g(x-at) +f(x,t) + int_0^t nabla f(x+(s-t)a, s) cdot (-a) ds \ + a cdot nabla g(x-at) + acdot int_0^t nabla f(x+(s-t)a, s) ds$$ $$= f(x, t)$$ as required



    However, I have no idea how these terms have appeared. In particular, I'm not sure why we have a grad appearing for $g$ and the $f(x, t)$ seems to come from nowhere...



    Any insight is appreciated.










    share|cite|improve this question











    $endgroup$















      0












      0








      0


      1



      $begingroup$



      We have $$u(x, t) = g(x-at, 0) + int_0^t f(x+a(s-t), s)ds$$ and want to show it satisfies the PDE $$partial_t u + a cdot Du = f$$




      The solution given goes as follows $$partial_t u + acdot Du = - a cdot nabla g(x-at) +f(x,t) + int_0^t nabla f(x+(s-t)a, s) cdot (-a) ds \ + a cdot nabla g(x-at) + acdot int_0^t nabla f(x+(s-t)a, s) ds$$ $$= f(x, t)$$ as required



      However, I have no idea how these terms have appeared. In particular, I'm not sure why we have a grad appearing for $g$ and the $f(x, t)$ seems to come from nowhere...



      Any insight is appreciated.










      share|cite|improve this question











      $endgroup$





      We have $$u(x, t) = g(x-at, 0) + int_0^t f(x+a(s-t), s)ds$$ and want to show it satisfies the PDE $$partial_t u + a cdot Du = f$$




      The solution given goes as follows $$partial_t u + acdot Du = - a cdot nabla g(x-at) +f(x,t) + int_0^t nabla f(x+(s-t)a, s) cdot (-a) ds \ + a cdot nabla g(x-at) + acdot int_0^t nabla f(x+(s-t)a, s) ds$$ $$= f(x, t)$$ as required



      However, I have no idea how these terms have appeared. In particular, I'm not sure why we have a grad appearing for $g$ and the $f(x, t)$ seems to come from nowhere...



      Any insight is appreciated.







      pde chain-rule transport-equation






      share|cite|improve this question















      share|cite|improve this question













      share|cite|improve this question




      share|cite|improve this question








      edited Jan 16 at 16:40









      Harry49

      8,47431344




      8,47431344










      asked Jan 16 at 8:26









      PhysicsMathsLovePhysicsMathsLove

      1,236415




      1,236415






















          3 Answers
          3






          active

          oldest

          votes


















          1












          $begingroup$

          The first three terms arise from the chain rule to calculate $u_t$. First, you dot-multiply the gradient of $g$ by the derivative of the function inside $g$ with respect to $t$, which is the vector $-a$, next you have to take the $t$-derivative of an integral that contains $t$ both in the limit of integration and in the integrand. For that you have to combine the fundamental theorem, setting $s=t$ in the integrand, and a "convective" term including the $t$ derivative of the integrand $-a$. The last two terms arise in a similar way, but now to take the $x$ - gradient of the integral with fixed limits you just take it on the integrand.



          It is just too long to write down all details...






          share|cite|improve this answer









          $endgroup$





















            0












            $begingroup$

            Hint: Use Leibniz's integral rule to show that



            begin{align}
            frac{partial}{partial t}int_0^t f(x+a(s-t),s) ds &= f(x+a(s-t),s)Bigvert_{s=t} - int_0^t frac{partial}{partial t}f(x+a(s-t),s) ds \
            &= f(x,t) - int_0^t (-a)f'(x+a(s-t),s) ds
            end{align}



            The gradient here just means the first derivative of a single variable, i.e.



            $$ frac{partial}{partial t} g(x-at) = -ag'(x-at) $$



            Try it your self with the remaining derivatives.






            share|cite|improve this answer









            $endgroup$





















              0












              $begingroup$

              Consider the function :
              $$u(x, t) = g(X, Y) + int_0^t f(Psi, Phi)ds$$
              where $$begin{cases}
              X=x-at & frac{partial X}{partial x}=1 quad & frac{partial X}{partial t}=-a\
              Y=0 \
              Psi=x+a(s-t) & frac{partial Psi}{partial x}=1quad & frac{partialPsi}{partial t}=-a\
              Phi=s end{cases}$$



              $frac{partial g}{partial x}=frac{partial g}{partial X}frac{partial X}{partial x}=frac{partial g}{partial X}$



              $frac{partial g}{partial t}=frac{partial g}{partial X}frac{partial X}{partial t}=-afrac{partial g}{partial X}$



              $frac{partial f}{partial x}=frac{partial f}{partial Psi}frac{partial Psi}{partial x}=frac{partial f}{partial Psi}$



              $frac{partial f}{partial t}=frac{partial f}{partial Psi}frac{partial Psi}{partial t}=-afrac{partial f}{partial Psi}$



              $frac{partial }{partial x}int_0^t f(Psi, Phi)ds =int_0^t frac{partial f}{partial x}ds =int_0^t frac{partial f}{partial psi}ds$



              $frac{partial }{partial t}int_0^t f(Psi,Phi)ds = left(f(Psi,Phi)right)_{s=t}+int_0^t frac{partial f}{partial t}ds = f(x,t)-aint_0^t frac{partial f}{partial Psi}ds $



              Because $f(Psi,Phi)=f(x,t)$ for $s=t$ :



              $Psi=x+a(s-t)=x+a(t-t)=xquad$ and $quadPhi=s=t$ .



              $$frac{partial u }{partial x}=frac{partial g }{partial x}+frac{partial }{partial x}int_0^t f(Psi, Phi)ds= frac{partial g}{partial X}+int_0^t frac{partial f}{partial Psi}ds$$



              $$frac{partial u }{partial t}=frac{partial g }{partial t}+frac{partial }{partial t}int_0^t f(Psi, Phi)ds=-afrac{partial g}{partial X} +f(x,t)-aint_0^t frac{partial f}{partial Psi}ds $$



              $$frac{partial u }{partial t}+afrac{partial u }{partial x}=
              left(-afrac{partial g}{partial X} +f(x,t)-aint_0^t frac{partial f}{partial Psi}ds right) +
              aleft( frac{partial g}{partial X}+int_0^t frac{partial f}{partial Psi}dsright)$$

              After simplification :
              $quadfrac{partial u }{partial t}+afrac{partial u }{partial x}=f(x,t)$



              Thus the function $quad u(x, t) = g(x-at, 0) + int_0^t f(x+a(s-t), s)dsquad$
              satisfies the PDE $quad partial_t u + a: partial_x u = f$ .






              share|cite|improve this answer











              $endgroup$














                Your Answer





                StackExchange.ifUsing("editor", function () {
                return StackExchange.using("mathjaxEditing", function () {
                StackExchange.MarkdownEditor.creationCallbacks.add(function (editor, postfix) {
                StackExchange.mathjaxEditing.prepareWmdForMathJax(editor, postfix, [["$", "$"], ["\\(","\\)"]]);
                });
                });
                }, "mathjax-editing");

                StackExchange.ready(function() {
                var channelOptions = {
                tags: "".split(" "),
                id: "69"
                };
                initTagRenderer("".split(" "), "".split(" "), channelOptions);

                StackExchange.using("externalEditor", function() {
                // Have to fire editor after snippets, if snippets enabled
                if (StackExchange.settings.snippets.snippetsEnabled) {
                StackExchange.using("snippets", function() {
                createEditor();
                });
                }
                else {
                createEditor();
                }
                });

                function createEditor() {
                StackExchange.prepareEditor({
                heartbeatType: 'answer',
                autoActivateHeartbeat: false,
                convertImagesToLinks: true,
                noModals: true,
                showLowRepImageUploadWarning: true,
                reputationToPostImages: 10,
                bindNavPrevention: true,
                postfix: "",
                imageUploader: {
                brandingHtml: "Powered by u003ca class="icon-imgur-white" href="https://imgur.com/"u003eu003c/au003e",
                contentPolicyHtml: "User contributions licensed under u003ca href="https://creativecommons.org/licenses/by-sa/3.0/"u003ecc by-sa 3.0 with attribution requiredu003c/au003e u003ca href="https://stackoverflow.com/legal/content-policy"u003e(content policy)u003c/au003e",
                allowUrls: true
                },
                noCode: true, onDemand: true,
                discardSelector: ".discard-answer"
                ,immediatelyShowMarkdownHelp:true
                });


                }
                });














                draft saved

                draft discarded


















                StackExchange.ready(
                function () {
                StackExchange.openid.initPostLogin('.new-post-login', 'https%3a%2f%2fmath.stackexchange.com%2fquestions%2f3075467%2fshowing-solution-satisfies-pde-chain-rule-help%23new-answer', 'question_page');
                }
                );

                Post as a guest















                Required, but never shown

























                3 Answers
                3






                active

                oldest

                votes








                3 Answers
                3






                active

                oldest

                votes









                active

                oldest

                votes






                active

                oldest

                votes









                1












                $begingroup$

                The first three terms arise from the chain rule to calculate $u_t$. First, you dot-multiply the gradient of $g$ by the derivative of the function inside $g$ with respect to $t$, which is the vector $-a$, next you have to take the $t$-derivative of an integral that contains $t$ both in the limit of integration and in the integrand. For that you have to combine the fundamental theorem, setting $s=t$ in the integrand, and a "convective" term including the $t$ derivative of the integrand $-a$. The last two terms arise in a similar way, but now to take the $x$ - gradient of the integral with fixed limits you just take it on the integrand.



                It is just too long to write down all details...






                share|cite|improve this answer









                $endgroup$


















                  1












                  $begingroup$

                  The first three terms arise from the chain rule to calculate $u_t$. First, you dot-multiply the gradient of $g$ by the derivative of the function inside $g$ with respect to $t$, which is the vector $-a$, next you have to take the $t$-derivative of an integral that contains $t$ both in the limit of integration and in the integrand. For that you have to combine the fundamental theorem, setting $s=t$ in the integrand, and a "convective" term including the $t$ derivative of the integrand $-a$. The last two terms arise in a similar way, but now to take the $x$ - gradient of the integral with fixed limits you just take it on the integrand.



                  It is just too long to write down all details...






                  share|cite|improve this answer









                  $endgroup$
















                    1












                    1








                    1





                    $begingroup$

                    The first three terms arise from the chain rule to calculate $u_t$. First, you dot-multiply the gradient of $g$ by the derivative of the function inside $g$ with respect to $t$, which is the vector $-a$, next you have to take the $t$-derivative of an integral that contains $t$ both in the limit of integration and in the integrand. For that you have to combine the fundamental theorem, setting $s=t$ in the integrand, and a "convective" term including the $t$ derivative of the integrand $-a$. The last two terms arise in a similar way, but now to take the $x$ - gradient of the integral with fixed limits you just take it on the integrand.



                    It is just too long to write down all details...






                    share|cite|improve this answer









                    $endgroup$



                    The first three terms arise from the chain rule to calculate $u_t$. First, you dot-multiply the gradient of $g$ by the derivative of the function inside $g$ with respect to $t$, which is the vector $-a$, next you have to take the $t$-derivative of an integral that contains $t$ both in the limit of integration and in the integrand. For that you have to combine the fundamental theorem, setting $s=t$ in the integrand, and a "convective" term including the $t$ derivative of the integrand $-a$. The last two terms arise in a similar way, but now to take the $x$ - gradient of the integral with fixed limits you just take it on the integrand.



                    It is just too long to write down all details...







                    share|cite|improve this answer












                    share|cite|improve this answer



                    share|cite|improve this answer










                    answered Jan 16 at 9:00









                    GReyesGReyes

                    2,33315




                    2,33315























                        0












                        $begingroup$

                        Hint: Use Leibniz's integral rule to show that



                        begin{align}
                        frac{partial}{partial t}int_0^t f(x+a(s-t),s) ds &= f(x+a(s-t),s)Bigvert_{s=t} - int_0^t frac{partial}{partial t}f(x+a(s-t),s) ds \
                        &= f(x,t) - int_0^t (-a)f'(x+a(s-t),s) ds
                        end{align}



                        The gradient here just means the first derivative of a single variable, i.e.



                        $$ frac{partial}{partial t} g(x-at) = -ag'(x-at) $$



                        Try it your self with the remaining derivatives.






                        share|cite|improve this answer









                        $endgroup$


















                          0












                          $begingroup$

                          Hint: Use Leibniz's integral rule to show that



                          begin{align}
                          frac{partial}{partial t}int_0^t f(x+a(s-t),s) ds &= f(x+a(s-t),s)Bigvert_{s=t} - int_0^t frac{partial}{partial t}f(x+a(s-t),s) ds \
                          &= f(x,t) - int_0^t (-a)f'(x+a(s-t),s) ds
                          end{align}



                          The gradient here just means the first derivative of a single variable, i.e.



                          $$ frac{partial}{partial t} g(x-at) = -ag'(x-at) $$



                          Try it your self with the remaining derivatives.






                          share|cite|improve this answer









                          $endgroup$
















                            0












                            0








                            0





                            $begingroup$

                            Hint: Use Leibniz's integral rule to show that



                            begin{align}
                            frac{partial}{partial t}int_0^t f(x+a(s-t),s) ds &= f(x+a(s-t),s)Bigvert_{s=t} - int_0^t frac{partial}{partial t}f(x+a(s-t),s) ds \
                            &= f(x,t) - int_0^t (-a)f'(x+a(s-t),s) ds
                            end{align}



                            The gradient here just means the first derivative of a single variable, i.e.



                            $$ frac{partial}{partial t} g(x-at) = -ag'(x-at) $$



                            Try it your self with the remaining derivatives.






                            share|cite|improve this answer









                            $endgroup$



                            Hint: Use Leibniz's integral rule to show that



                            begin{align}
                            frac{partial}{partial t}int_0^t f(x+a(s-t),s) ds &= f(x+a(s-t),s)Bigvert_{s=t} - int_0^t frac{partial}{partial t}f(x+a(s-t),s) ds \
                            &= f(x,t) - int_0^t (-a)f'(x+a(s-t),s) ds
                            end{align}



                            The gradient here just means the first derivative of a single variable, i.e.



                            $$ frac{partial}{partial t} g(x-at) = -ag'(x-at) $$



                            Try it your self with the remaining derivatives.







                            share|cite|improve this answer












                            share|cite|improve this answer



                            share|cite|improve this answer










                            answered Jan 16 at 12:22









                            DylanDylan

                            14.2k31127




                            14.2k31127























                                0












                                $begingroup$

                                Consider the function :
                                $$u(x, t) = g(X, Y) + int_0^t f(Psi, Phi)ds$$
                                where $$begin{cases}
                                X=x-at & frac{partial X}{partial x}=1 quad & frac{partial X}{partial t}=-a\
                                Y=0 \
                                Psi=x+a(s-t) & frac{partial Psi}{partial x}=1quad & frac{partialPsi}{partial t}=-a\
                                Phi=s end{cases}$$



                                $frac{partial g}{partial x}=frac{partial g}{partial X}frac{partial X}{partial x}=frac{partial g}{partial X}$



                                $frac{partial g}{partial t}=frac{partial g}{partial X}frac{partial X}{partial t}=-afrac{partial g}{partial X}$



                                $frac{partial f}{partial x}=frac{partial f}{partial Psi}frac{partial Psi}{partial x}=frac{partial f}{partial Psi}$



                                $frac{partial f}{partial t}=frac{partial f}{partial Psi}frac{partial Psi}{partial t}=-afrac{partial f}{partial Psi}$



                                $frac{partial }{partial x}int_0^t f(Psi, Phi)ds =int_0^t frac{partial f}{partial x}ds =int_0^t frac{partial f}{partial psi}ds$



                                $frac{partial }{partial t}int_0^t f(Psi,Phi)ds = left(f(Psi,Phi)right)_{s=t}+int_0^t frac{partial f}{partial t}ds = f(x,t)-aint_0^t frac{partial f}{partial Psi}ds $



                                Because $f(Psi,Phi)=f(x,t)$ for $s=t$ :



                                $Psi=x+a(s-t)=x+a(t-t)=xquad$ and $quadPhi=s=t$ .



                                $$frac{partial u }{partial x}=frac{partial g }{partial x}+frac{partial }{partial x}int_0^t f(Psi, Phi)ds= frac{partial g}{partial X}+int_0^t frac{partial f}{partial Psi}ds$$



                                $$frac{partial u }{partial t}=frac{partial g }{partial t}+frac{partial }{partial t}int_0^t f(Psi, Phi)ds=-afrac{partial g}{partial X} +f(x,t)-aint_0^t frac{partial f}{partial Psi}ds $$



                                $$frac{partial u }{partial t}+afrac{partial u }{partial x}=
                                left(-afrac{partial g}{partial X} +f(x,t)-aint_0^t frac{partial f}{partial Psi}ds right) +
                                aleft( frac{partial g}{partial X}+int_0^t frac{partial f}{partial Psi}dsright)$$

                                After simplification :
                                $quadfrac{partial u }{partial t}+afrac{partial u }{partial x}=f(x,t)$



                                Thus the function $quad u(x, t) = g(x-at, 0) + int_0^t f(x+a(s-t), s)dsquad$
                                satisfies the PDE $quad partial_t u + a: partial_x u = f$ .






                                share|cite|improve this answer











                                $endgroup$


















                                  0












                                  $begingroup$

                                  Consider the function :
                                  $$u(x, t) = g(X, Y) + int_0^t f(Psi, Phi)ds$$
                                  where $$begin{cases}
                                  X=x-at & frac{partial X}{partial x}=1 quad & frac{partial X}{partial t}=-a\
                                  Y=0 \
                                  Psi=x+a(s-t) & frac{partial Psi}{partial x}=1quad & frac{partialPsi}{partial t}=-a\
                                  Phi=s end{cases}$$



                                  $frac{partial g}{partial x}=frac{partial g}{partial X}frac{partial X}{partial x}=frac{partial g}{partial X}$



                                  $frac{partial g}{partial t}=frac{partial g}{partial X}frac{partial X}{partial t}=-afrac{partial g}{partial X}$



                                  $frac{partial f}{partial x}=frac{partial f}{partial Psi}frac{partial Psi}{partial x}=frac{partial f}{partial Psi}$



                                  $frac{partial f}{partial t}=frac{partial f}{partial Psi}frac{partial Psi}{partial t}=-afrac{partial f}{partial Psi}$



                                  $frac{partial }{partial x}int_0^t f(Psi, Phi)ds =int_0^t frac{partial f}{partial x}ds =int_0^t frac{partial f}{partial psi}ds$



                                  $frac{partial }{partial t}int_0^t f(Psi,Phi)ds = left(f(Psi,Phi)right)_{s=t}+int_0^t frac{partial f}{partial t}ds = f(x,t)-aint_0^t frac{partial f}{partial Psi}ds $



                                  Because $f(Psi,Phi)=f(x,t)$ for $s=t$ :



                                  $Psi=x+a(s-t)=x+a(t-t)=xquad$ and $quadPhi=s=t$ .



                                  $$frac{partial u }{partial x}=frac{partial g }{partial x}+frac{partial }{partial x}int_0^t f(Psi, Phi)ds= frac{partial g}{partial X}+int_0^t frac{partial f}{partial Psi}ds$$



                                  $$frac{partial u }{partial t}=frac{partial g }{partial t}+frac{partial }{partial t}int_0^t f(Psi, Phi)ds=-afrac{partial g}{partial X} +f(x,t)-aint_0^t frac{partial f}{partial Psi}ds $$



                                  $$frac{partial u }{partial t}+afrac{partial u }{partial x}=
                                  left(-afrac{partial g}{partial X} +f(x,t)-aint_0^t frac{partial f}{partial Psi}ds right) +
                                  aleft( frac{partial g}{partial X}+int_0^t frac{partial f}{partial Psi}dsright)$$

                                  After simplification :
                                  $quadfrac{partial u }{partial t}+afrac{partial u }{partial x}=f(x,t)$



                                  Thus the function $quad u(x, t) = g(x-at, 0) + int_0^t f(x+a(s-t), s)dsquad$
                                  satisfies the PDE $quad partial_t u + a: partial_x u = f$ .






                                  share|cite|improve this answer











                                  $endgroup$
















                                    0












                                    0








                                    0





                                    $begingroup$

                                    Consider the function :
                                    $$u(x, t) = g(X, Y) + int_0^t f(Psi, Phi)ds$$
                                    where $$begin{cases}
                                    X=x-at & frac{partial X}{partial x}=1 quad & frac{partial X}{partial t}=-a\
                                    Y=0 \
                                    Psi=x+a(s-t) & frac{partial Psi}{partial x}=1quad & frac{partialPsi}{partial t}=-a\
                                    Phi=s end{cases}$$



                                    $frac{partial g}{partial x}=frac{partial g}{partial X}frac{partial X}{partial x}=frac{partial g}{partial X}$



                                    $frac{partial g}{partial t}=frac{partial g}{partial X}frac{partial X}{partial t}=-afrac{partial g}{partial X}$



                                    $frac{partial f}{partial x}=frac{partial f}{partial Psi}frac{partial Psi}{partial x}=frac{partial f}{partial Psi}$



                                    $frac{partial f}{partial t}=frac{partial f}{partial Psi}frac{partial Psi}{partial t}=-afrac{partial f}{partial Psi}$



                                    $frac{partial }{partial x}int_0^t f(Psi, Phi)ds =int_0^t frac{partial f}{partial x}ds =int_0^t frac{partial f}{partial psi}ds$



                                    $frac{partial }{partial t}int_0^t f(Psi,Phi)ds = left(f(Psi,Phi)right)_{s=t}+int_0^t frac{partial f}{partial t}ds = f(x,t)-aint_0^t frac{partial f}{partial Psi}ds $



                                    Because $f(Psi,Phi)=f(x,t)$ for $s=t$ :



                                    $Psi=x+a(s-t)=x+a(t-t)=xquad$ and $quadPhi=s=t$ .



                                    $$frac{partial u }{partial x}=frac{partial g }{partial x}+frac{partial }{partial x}int_0^t f(Psi, Phi)ds= frac{partial g}{partial X}+int_0^t frac{partial f}{partial Psi}ds$$



                                    $$frac{partial u }{partial t}=frac{partial g }{partial t}+frac{partial }{partial t}int_0^t f(Psi, Phi)ds=-afrac{partial g}{partial X} +f(x,t)-aint_0^t frac{partial f}{partial Psi}ds $$



                                    $$frac{partial u }{partial t}+afrac{partial u }{partial x}=
                                    left(-afrac{partial g}{partial X} +f(x,t)-aint_0^t frac{partial f}{partial Psi}ds right) +
                                    aleft( frac{partial g}{partial X}+int_0^t frac{partial f}{partial Psi}dsright)$$

                                    After simplification :
                                    $quadfrac{partial u }{partial t}+afrac{partial u }{partial x}=f(x,t)$



                                    Thus the function $quad u(x, t) = g(x-at, 0) + int_0^t f(x+a(s-t), s)dsquad$
                                    satisfies the PDE $quad partial_t u + a: partial_x u = f$ .






                                    share|cite|improve this answer











                                    $endgroup$



                                    Consider the function :
                                    $$u(x, t) = g(X, Y) + int_0^t f(Psi, Phi)ds$$
                                    where $$begin{cases}
                                    X=x-at & frac{partial X}{partial x}=1 quad & frac{partial X}{partial t}=-a\
                                    Y=0 \
                                    Psi=x+a(s-t) & frac{partial Psi}{partial x}=1quad & frac{partialPsi}{partial t}=-a\
                                    Phi=s end{cases}$$



                                    $frac{partial g}{partial x}=frac{partial g}{partial X}frac{partial X}{partial x}=frac{partial g}{partial X}$



                                    $frac{partial g}{partial t}=frac{partial g}{partial X}frac{partial X}{partial t}=-afrac{partial g}{partial X}$



                                    $frac{partial f}{partial x}=frac{partial f}{partial Psi}frac{partial Psi}{partial x}=frac{partial f}{partial Psi}$



                                    $frac{partial f}{partial t}=frac{partial f}{partial Psi}frac{partial Psi}{partial t}=-afrac{partial f}{partial Psi}$



                                    $frac{partial }{partial x}int_0^t f(Psi, Phi)ds =int_0^t frac{partial f}{partial x}ds =int_0^t frac{partial f}{partial psi}ds$



                                    $frac{partial }{partial t}int_0^t f(Psi,Phi)ds = left(f(Psi,Phi)right)_{s=t}+int_0^t frac{partial f}{partial t}ds = f(x,t)-aint_0^t frac{partial f}{partial Psi}ds $



                                    Because $f(Psi,Phi)=f(x,t)$ for $s=t$ :



                                    $Psi=x+a(s-t)=x+a(t-t)=xquad$ and $quadPhi=s=t$ .



                                    $$frac{partial u }{partial x}=frac{partial g }{partial x}+frac{partial }{partial x}int_0^t f(Psi, Phi)ds= frac{partial g}{partial X}+int_0^t frac{partial f}{partial Psi}ds$$



                                    $$frac{partial u }{partial t}=frac{partial g }{partial t}+frac{partial }{partial t}int_0^t f(Psi, Phi)ds=-afrac{partial g}{partial X} +f(x,t)-aint_0^t frac{partial f}{partial Psi}ds $$



                                    $$frac{partial u }{partial t}+afrac{partial u }{partial x}=
                                    left(-afrac{partial g}{partial X} +f(x,t)-aint_0^t frac{partial f}{partial Psi}ds right) +
                                    aleft( frac{partial g}{partial X}+int_0^t frac{partial f}{partial Psi}dsright)$$

                                    After simplification :
                                    $quadfrac{partial u }{partial t}+afrac{partial u }{partial x}=f(x,t)$



                                    Thus the function $quad u(x, t) = g(x-at, 0) + int_0^t f(x+a(s-t), s)dsquad$
                                    satisfies the PDE $quad partial_t u + a: partial_x u = f$ .







                                    share|cite|improve this answer














                                    share|cite|improve this answer



                                    share|cite|improve this answer








                                    edited Jan 16 at 17:01

























                                    answered Jan 16 at 14:00









                                    JJacquelinJJacquelin

                                    45.3k21856




                                    45.3k21856






























                                        draft saved

                                        draft discarded




















































                                        Thanks for contributing an answer to Mathematics Stack Exchange!


                                        • Please be sure to answer the question. Provide details and share your research!

                                        But avoid



                                        • Asking for help, clarification, or responding to other answers.

                                        • Making statements based on opinion; back them up with references or personal experience.


                                        Use MathJax to format equations. MathJax reference.


                                        To learn more, see our tips on writing great answers.




                                        draft saved


                                        draft discarded














                                        StackExchange.ready(
                                        function () {
                                        StackExchange.openid.initPostLogin('.new-post-login', 'https%3a%2f%2fmath.stackexchange.com%2fquestions%2f3075467%2fshowing-solution-satisfies-pde-chain-rule-help%23new-answer', 'question_page');
                                        }
                                        );

                                        Post as a guest















                                        Required, but never shown





















































                                        Required, but never shown














                                        Required, but never shown












                                        Required, but never shown







                                        Required, but never shown

































                                        Required, but never shown














                                        Required, but never shown












                                        Required, but never shown







                                        Required, but never shown







                                        Popular posts from this blog

                                        Human spaceflight

                                        Can not write log (Is /dev/pts mounted?) - openpty in Ubuntu-on-Windows?

                                        File:DeusFollowingSea.jpg